LSAT and Law School Admissions Forum

Get expert LSAT preparation and law school admissions advice from PowerScore Test Preparation.

User avatar
 Dave Killoran
PowerScore Staff
  • PowerScore Staff
  • Posts: 5850
  • Joined: Mar 25, 2011
|
#85630
Complete Question Explanation

(The complete setup for this game can be found here: viewtopic.php?f=386&t=3729)

The correct answer choice is (E)

The condition in the question stem creates the following sequence:
Capture5.PNG
However, because K, S, and F are all utensils, they must be washed consecutively, and therefore they
must form a block:
Capture6.PNG
Accordingly, in Template 2 neither answer choice (A) nor answer choice (B) can be true. Answer
choices (C) and (D) can be eliminated because they conform to neither template. Thus, answer
choice (E) is proven correct
by process of elimination, and it could occur under Template 2.

Get the most out of your LSAT Prep Plus subscription.

Analyze and track your performance with our Testing and Analytics Package.